Value of the double integral of $int_0^1int_1^3 (4x^3-27x^2y^2)dydx$On the convergence of an improper integralIntegral$int_1^infty log log left(xright)fracdx1-x+x^2=frac2pisqrt 3left(frac56log (2pi)-log Gamma frac16right)$Find the second derivative of a double integralIntegral with Limit and ParametersHow to get the asymptotic form of this oscilatting integral?On the convergence of $-int_0^1left(int_1^infty left(frac1zeta(x/y)-1right)dxright)dy$How to prove the following ineqiulity : $expleft(int_0^t f(s)ds right) le 1+ int_0^t e^max(1,s)f(s)ds$Solid volume using double integralJustify an approximation of the first few decimals of $int_1^infty(int_0^1fracdx-1+x^3+y^2)dy$Double integral over triangle, what bounds should be chosen?

How can I raise concerns with a new DM about XP splitting?

Does "Dominei" mean something?

Books on the History of math research at European universities

Is infinity mathematically observable?

Can somebody explain Brexit in a few child-proof sentences?

How can I successfully establish a nationwide combat training program for a large country?

Hostile work environment after whistle-blowing on coworker and our boss. What do I do?

Identify a stage play about a VR experience in which participants are encouraged to simulate performing horrific activities

Can a Gentile theist be saved?

In Star Trek IV, why did the Bounty go back to a time when whales were already rare?

Invariance of results when scaling explanatory variables in logistic regression, is there a proof?

Why is delta-v is the most useful quantity for planning space travel?

Visiting the UK as unmarried couple

Can I Retrieve Email Addresses from BCC?

Is there enough fresh water in the world to eradicate the drinking water crisis?

For airliners, what prevents wing strikes on landing in bad weather?

Freedom of speech and where it applies

Can I use my Chinese passport to enter China after I acquired another citizenship?

The most efficient algorithm to find all possible integer pairs which sum to a given integer

Reply ‘no position’ while the job posting is still there (‘HiWi’ position in Germany)

Word describing multiple paths to the same abstract outcome

Installing PowerShell on 32-bit Kali OS fails

Is it okay / does it make sense for another player to join a running game of Munchkin?

Is there a good way to store credentials outside of a password manager?



Value of the double integral of $int_0^1int_1^3 (4x^3-27x^2y^2)dydx$


On the convergence of an improper integralIntegral$int_1^infty log log left(xright)fracdx1-x+x^2=frac2pisqrt 3left(frac56log (2pi)-log Gamma frac16right)$Find the second derivative of a double integralIntegral with Limit and ParametersHow to get the asymptotic form of this oscilatting integral?On the convergence of $-int_0^1left(int_1^infty left(frac1zeta(x/y)-1right)dxright)dy$How to prove the following ineqiulity : $expleft(int_0^t f(s)ds right) le 1+ int_0^t e^max(1,s)f(s)ds$Solid volume using double integralJustify an approximation of the first few decimals of $int_1^infty(int_0^1fracdx-1+x^3+y^2)dy$Double integral over triangle, what bounds should be chosen?













1












$begingroup$


$$int_0^1int_1^3left(4x^3-27x^2y^2right)dy,dx$$



$$int_0^1left(4x^3y-9x^2y^3right)Big|_1^3,dx$$



$$int_0^1left(12x^3-243x^2-4x^3+9x^2right)dx$$



$$int_0^1left(8x^3-234x^2right)dx$$



$$left(2x^4-78x^3right)Big|_0^1$$



Of course this gives a negative answer and is supposedly wrong, but I don't see any problem with my work. I know for a fact that when you take the derivative with respect to $y$, you will get $4x^3y-9x^2y^3$, and I know for a fact that when you plug in $3$ you will get $3^3$ which is $27$ and multiply that by $9$ and you get $243$! These supposed arithmetic errors are killing me! I don't even see any in this problem.










share|cite|improve this question











$endgroup$











  • $begingroup$
    What is the region and the function?
    $endgroup$
    – AlgorithmsX
    Dec 3 '16 at 20:34






  • 3




    $begingroup$
    You're solving the integral correctly, W|A confirms it as well. If that's not the right answer, perhaps the problem is in setting up the integral.
    $endgroup$
    – 2012ssohn
    Dec 3 '16 at 20:35










  • $begingroup$
    To stop banging your head against your wall, you might want to compute the double integral the other way round, that is, integrating first with respect to $x$ then with respect to $y$. That is, start with $int_0^1(4x^3-27x^2y^2)dx=left.x^4-9x^3y^2right|_0^1=1-9y^2$ hence your double integral is $int_1^3(1-9y^2)dy=left.y-3y^3right|_1^3=3-3cdot3^3-1+3=-76$. This does not prove your first computation is correct but this definitely suggests that it might be so...
    $endgroup$
    – Did
    Dec 3 '16 at 20:41











  • $begingroup$
    Are we supposed to be integrating over the rectangle bound by the $y$-axis and the lines $x=1$, $y=1$, and $y=3$? If not, then I'm afraid you've set up the integral incorrectly.
    $endgroup$
    – teadawg1337
    Dec 3 '16 at 20:58















1












$begingroup$


$$int_0^1int_1^3left(4x^3-27x^2y^2right)dy,dx$$



$$int_0^1left(4x^3y-9x^2y^3right)Big|_1^3,dx$$



$$int_0^1left(12x^3-243x^2-4x^3+9x^2right)dx$$



$$int_0^1left(8x^3-234x^2right)dx$$



$$left(2x^4-78x^3right)Big|_0^1$$



Of course this gives a negative answer and is supposedly wrong, but I don't see any problem with my work. I know for a fact that when you take the derivative with respect to $y$, you will get $4x^3y-9x^2y^3$, and I know for a fact that when you plug in $3$ you will get $3^3$ which is $27$ and multiply that by $9$ and you get $243$! These supposed arithmetic errors are killing me! I don't even see any in this problem.










share|cite|improve this question











$endgroup$











  • $begingroup$
    What is the region and the function?
    $endgroup$
    – AlgorithmsX
    Dec 3 '16 at 20:34






  • 3




    $begingroup$
    You're solving the integral correctly, W|A confirms it as well. If that's not the right answer, perhaps the problem is in setting up the integral.
    $endgroup$
    – 2012ssohn
    Dec 3 '16 at 20:35










  • $begingroup$
    To stop banging your head against your wall, you might want to compute the double integral the other way round, that is, integrating first with respect to $x$ then with respect to $y$. That is, start with $int_0^1(4x^3-27x^2y^2)dx=left.x^4-9x^3y^2right|_0^1=1-9y^2$ hence your double integral is $int_1^3(1-9y^2)dy=left.y-3y^3right|_1^3=3-3cdot3^3-1+3=-76$. This does not prove your first computation is correct but this definitely suggests that it might be so...
    $endgroup$
    – Did
    Dec 3 '16 at 20:41











  • $begingroup$
    Are we supposed to be integrating over the rectangle bound by the $y$-axis and the lines $x=1$, $y=1$, and $y=3$? If not, then I'm afraid you've set up the integral incorrectly.
    $endgroup$
    – teadawg1337
    Dec 3 '16 at 20:58













1












1








1





$begingroup$


$$int_0^1int_1^3left(4x^3-27x^2y^2right)dy,dx$$



$$int_0^1left(4x^3y-9x^2y^3right)Big|_1^3,dx$$



$$int_0^1left(12x^3-243x^2-4x^3+9x^2right)dx$$



$$int_0^1left(8x^3-234x^2right)dx$$



$$left(2x^4-78x^3right)Big|_0^1$$



Of course this gives a negative answer and is supposedly wrong, but I don't see any problem with my work. I know for a fact that when you take the derivative with respect to $y$, you will get $4x^3y-9x^2y^3$, and I know for a fact that when you plug in $3$ you will get $3^3$ which is $27$ and multiply that by $9$ and you get $243$! These supposed arithmetic errors are killing me! I don't even see any in this problem.










share|cite|improve this question











$endgroup$




$$int_0^1int_1^3left(4x^3-27x^2y^2right)dy,dx$$



$$int_0^1left(4x^3y-9x^2y^3right)Big|_1^3,dx$$



$$int_0^1left(12x^3-243x^2-4x^3+9x^2right)dx$$



$$int_0^1left(8x^3-234x^2right)dx$$



$$left(2x^4-78x^3right)Big|_0^1$$



Of course this gives a negative answer and is supposedly wrong, but I don't see any problem with my work. I know for a fact that when you take the derivative with respect to $y$, you will get $4x^3y-9x^2y^3$, and I know for a fact that when you plug in $3$ you will get $3^3$ which is $27$ and multiply that by $9$ and you get $243$! These supposed arithmetic errors are killing me! I don't even see any in this problem.







integration






share|cite|improve this question















share|cite|improve this question













share|cite|improve this question




share|cite|improve this question








edited Mar 16 at 21:39









Robert Howard

2,2383935




2,2383935










asked Dec 3 '16 at 20:30









K. GibsonK. Gibson

54621027




54621027











  • $begingroup$
    What is the region and the function?
    $endgroup$
    – AlgorithmsX
    Dec 3 '16 at 20:34






  • 3




    $begingroup$
    You're solving the integral correctly, W|A confirms it as well. If that's not the right answer, perhaps the problem is in setting up the integral.
    $endgroup$
    – 2012ssohn
    Dec 3 '16 at 20:35










  • $begingroup$
    To stop banging your head against your wall, you might want to compute the double integral the other way round, that is, integrating first with respect to $x$ then with respect to $y$. That is, start with $int_0^1(4x^3-27x^2y^2)dx=left.x^4-9x^3y^2right|_0^1=1-9y^2$ hence your double integral is $int_1^3(1-9y^2)dy=left.y-3y^3right|_1^3=3-3cdot3^3-1+3=-76$. This does not prove your first computation is correct but this definitely suggests that it might be so...
    $endgroup$
    – Did
    Dec 3 '16 at 20:41











  • $begingroup$
    Are we supposed to be integrating over the rectangle bound by the $y$-axis and the lines $x=1$, $y=1$, and $y=3$? If not, then I'm afraid you've set up the integral incorrectly.
    $endgroup$
    – teadawg1337
    Dec 3 '16 at 20:58
















  • $begingroup$
    What is the region and the function?
    $endgroup$
    – AlgorithmsX
    Dec 3 '16 at 20:34






  • 3




    $begingroup$
    You're solving the integral correctly, W|A confirms it as well. If that's not the right answer, perhaps the problem is in setting up the integral.
    $endgroup$
    – 2012ssohn
    Dec 3 '16 at 20:35










  • $begingroup$
    To stop banging your head against your wall, you might want to compute the double integral the other way round, that is, integrating first with respect to $x$ then with respect to $y$. That is, start with $int_0^1(4x^3-27x^2y^2)dx=left.x^4-9x^3y^2right|_0^1=1-9y^2$ hence your double integral is $int_1^3(1-9y^2)dy=left.y-3y^3right|_1^3=3-3cdot3^3-1+3=-76$. This does not prove your first computation is correct but this definitely suggests that it might be so...
    $endgroup$
    – Did
    Dec 3 '16 at 20:41











  • $begingroup$
    Are we supposed to be integrating over the rectangle bound by the $y$-axis and the lines $x=1$, $y=1$, and $y=3$? If not, then I'm afraid you've set up the integral incorrectly.
    $endgroup$
    – teadawg1337
    Dec 3 '16 at 20:58















$begingroup$
What is the region and the function?
$endgroup$
– AlgorithmsX
Dec 3 '16 at 20:34




$begingroup$
What is the region and the function?
$endgroup$
– AlgorithmsX
Dec 3 '16 at 20:34




3




3




$begingroup$
You're solving the integral correctly, W|A confirms it as well. If that's not the right answer, perhaps the problem is in setting up the integral.
$endgroup$
– 2012ssohn
Dec 3 '16 at 20:35




$begingroup$
You're solving the integral correctly, W|A confirms it as well. If that's not the right answer, perhaps the problem is in setting up the integral.
$endgroup$
– 2012ssohn
Dec 3 '16 at 20:35












$begingroup$
To stop banging your head against your wall, you might want to compute the double integral the other way round, that is, integrating first with respect to $x$ then with respect to $y$. That is, start with $int_0^1(4x^3-27x^2y^2)dx=left.x^4-9x^3y^2right|_0^1=1-9y^2$ hence your double integral is $int_1^3(1-9y^2)dy=left.y-3y^3right|_1^3=3-3cdot3^3-1+3=-76$. This does not prove your first computation is correct but this definitely suggests that it might be so...
$endgroup$
– Did
Dec 3 '16 at 20:41





$begingroup$
To stop banging your head against your wall, you might want to compute the double integral the other way round, that is, integrating first with respect to $x$ then with respect to $y$. That is, start with $int_0^1(4x^3-27x^2y^2)dx=left.x^4-9x^3y^2right|_0^1=1-9y^2$ hence your double integral is $int_1^3(1-9y^2)dy=left.y-3y^3right|_1^3=3-3cdot3^3-1+3=-76$. This does not prove your first computation is correct but this definitely suggests that it might be so...
$endgroup$
– Did
Dec 3 '16 at 20:41













$begingroup$
Are we supposed to be integrating over the rectangle bound by the $y$-axis and the lines $x=1$, $y=1$, and $y=3$? If not, then I'm afraid you've set up the integral incorrectly.
$endgroup$
– teadawg1337
Dec 3 '16 at 20:58




$begingroup$
Are we supposed to be integrating over the rectangle bound by the $y$-axis and the lines $x=1$, $y=1$, and $y=3$? If not, then I'm afraid you've set up the integral incorrectly.
$endgroup$
– teadawg1337
Dec 3 '16 at 20:58










1 Answer
1






active

oldest

votes


















1












$begingroup$

As the commenters said, you've evaluated the integral correctly. Every tool I use to check it confirms that the answer is $-76$. If that's not the answer you were looking for, you must have set up the integral incorrectly.






share|cite|improve this answer











$endgroup$












    Your Answer





    StackExchange.ifUsing("editor", function ()
    return StackExchange.using("mathjaxEditing", function ()
    StackExchange.MarkdownEditor.creationCallbacks.add(function (editor, postfix)
    StackExchange.mathjaxEditing.prepareWmdForMathJax(editor, postfix, [["$", "$"], ["\\(","\\)"]]);
    );
    );
    , "mathjax-editing");

    StackExchange.ready(function()
    var channelOptions =
    tags: "".split(" "),
    id: "69"
    ;
    initTagRenderer("".split(" "), "".split(" "), channelOptions);

    StackExchange.using("externalEditor", function()
    // Have to fire editor after snippets, if snippets enabled
    if (StackExchange.settings.snippets.snippetsEnabled)
    StackExchange.using("snippets", function()
    createEditor();
    );

    else
    createEditor();

    );

    function createEditor()
    StackExchange.prepareEditor(
    heartbeatType: 'answer',
    autoActivateHeartbeat: false,
    convertImagesToLinks: true,
    noModals: true,
    showLowRepImageUploadWarning: true,
    reputationToPostImages: 10,
    bindNavPrevention: true,
    postfix: "",
    imageUploader:
    brandingHtml: "Powered by u003ca class="icon-imgur-white" href="https://imgur.com/"u003eu003c/au003e",
    contentPolicyHtml: "User contributions licensed under u003ca href="https://creativecommons.org/licenses/by-sa/3.0/"u003ecc by-sa 3.0 with attribution requiredu003c/au003e u003ca href="https://stackoverflow.com/legal/content-policy"u003e(content policy)u003c/au003e",
    allowUrls: true
    ,
    noCode: true, onDemand: true,
    discardSelector: ".discard-answer"
    ,immediatelyShowMarkdownHelp:true
    );



    );













    draft saved

    draft discarded


















    StackExchange.ready(
    function ()
    StackExchange.openid.initPostLogin('.new-post-login', 'https%3a%2f%2fmath.stackexchange.com%2fquestions%2f2042311%2fvalue-of-the-double-integral-of-int-01-int-13-4x3-27x2y2dydx%23new-answer', 'question_page');

    );

    Post as a guest















    Required, but never shown

























    1 Answer
    1






    active

    oldest

    votes








    1 Answer
    1






    active

    oldest

    votes









    active

    oldest

    votes






    active

    oldest

    votes









    1












    $begingroup$

    As the commenters said, you've evaluated the integral correctly. Every tool I use to check it confirms that the answer is $-76$. If that's not the answer you were looking for, you must have set up the integral incorrectly.






    share|cite|improve this answer











    $endgroup$

















      1












      $begingroup$

      As the commenters said, you've evaluated the integral correctly. Every tool I use to check it confirms that the answer is $-76$. If that's not the answer you were looking for, you must have set up the integral incorrectly.






      share|cite|improve this answer











      $endgroup$















        1












        1








        1





        $begingroup$

        As the commenters said, you've evaluated the integral correctly. Every tool I use to check it confirms that the answer is $-76$. If that's not the answer you were looking for, you must have set up the integral incorrectly.






        share|cite|improve this answer











        $endgroup$



        As the commenters said, you've evaluated the integral correctly. Every tool I use to check it confirms that the answer is $-76$. If that's not the answer you were looking for, you must have set up the integral incorrectly.







        share|cite|improve this answer














        share|cite|improve this answer



        share|cite|improve this answer








        answered Mar 16 at 21:47


























        community wiki





        Robert Howard




























            draft saved

            draft discarded
















































            Thanks for contributing an answer to Mathematics Stack Exchange!


            • Please be sure to answer the question. Provide details and share your research!

            But avoid


            • Asking for help, clarification, or responding to other answers.

            • Making statements based on opinion; back them up with references or personal experience.

            Use MathJax to format equations. MathJax reference.


            To learn more, see our tips on writing great answers.




            draft saved


            draft discarded














            StackExchange.ready(
            function ()
            StackExchange.openid.initPostLogin('.new-post-login', 'https%3a%2f%2fmath.stackexchange.com%2fquestions%2f2042311%2fvalue-of-the-double-integral-of-int-01-int-13-4x3-27x2y2dydx%23new-answer', 'question_page');

            );

            Post as a guest















            Required, but never shown





















































            Required, but never shown














            Required, but never shown












            Required, but never shown







            Required, but never shown

































            Required, but never shown














            Required, but never shown












            Required, but never shown







            Required, but never shown







            Popular posts from this blog

            Lowndes Grove History Architecture References Navigation menu32°48′6″N 79°57′58″W / 32.80167°N 79.96611°W / 32.80167; -79.9661132°48′6″N 79°57′58″W / 32.80167°N 79.96611°W / 32.80167; -79.9661178002500"National Register Information System"Historic houses of South Carolina"Lowndes Grove""+32° 48' 6.00", −79° 57' 58.00""Lowndes Grove, Charleston County (260 St. Margaret St., Charleston)""Lowndes Grove"The Charleston ExpositionIt Happened in South Carolina"Lowndes Grove (House), Saint Margaret Street & Sixth Avenue, Charleston, Charleston County, SC(Photographs)"Plantations of the Carolina Low Countrye

            random experiment with two different functions on unit interval Announcing the arrival of Valued Associate #679: Cesar Manara Planned maintenance scheduled April 23, 2019 at 00:00UTC (8:00pm US/Eastern)Random variable and probability space notionsRandom Walk with EdgesFinding functions where the increase over a random interval is Poisson distributedNumber of days until dayCan an observed event in fact be of zero probability?Unit random processmodels of coins and uniform distributionHow to get the number of successes given $n$ trials , probability $P$ and a random variable $X$Absorbing Markov chain in a computer. Is “almost every” turned into always convergence in computer executions?Stopped random walk is not uniformly integrable

            How should I support this large drywall patch? Planned maintenance scheduled April 23, 2019 at 00:00UTC (8:00pm US/Eastern) Announcing the arrival of Valued Associate #679: Cesar Manara Unicorn Meta Zoo #1: Why another podcast?How do I cover large gaps in drywall?How do I keep drywall around a patch from crumbling?Can I glue a second layer of drywall?How to patch long strip on drywall?Large drywall patch: how to avoid bulging seams?Drywall Mesh Patch vs. Bulge? To remove or not to remove?How to fix this drywall job?Prep drywall before backsplashWhat's the best way to fix this horrible drywall patch job?Drywall patching using 3M Patch Plus Primer